PT63.S3.Q24 - physician: the rise in blood pressure

_kizilbash_kizilbash Member
edited December 2017 in Logical Reasoning 62 karma

Hi everyone!
Can someone please help me with this question. I can't understand why C is wrong at all. I got it down to A and C and ended up picking C.
cant C be correct because if drinking one glass doesn't cause deficiency in vitamin D, that means there would be enough calcium absorbed by an aging persons body for them to lower their blood pressure.
thoughts?

https://7sage.com/lsat_explanations/lsat-63-section-3-question-24/

Comments

  • akistotleakistotle Member 🍌🍌
    9377 karma

    https://7sage.com/lsat_explanations/lsat-63-section-3-question-24/

    The problem with (C) is that it isn't enough. Milk has to do more than "not contributing" to the deficiency of vitamin D. Milk has to contribute to old people getting active vitamin D.

    I think (B) and (C) are necessary assumptions, but not sufficient assumptions.

    [Stimulus]
    P: Deficiency of active form of vitamin D =cause=> Calcium deficiency =cause=> ↑Blood pressure

    P: One glass of milk per day can make up for calcium deficiency
    ————
    C: By drinking milk, some older people can ↓Blood pressure

    SA: Milk has active form of vitamin D

  • lTexlawzlTexlawz Free Trial Member
    277 karma

    The question stem indicates that is a sufficient assumption. Let's look at the stem for a moment.

    It says "The physician's conclusion is properly drawn if which one of the follow is assumed." Any time you see properly drawn, you think the answer choice should reflect sufficient. Key words like

    when, whenever, every, all, any, people who, in order to,some,most,Not all,at least in the short paragraph.

    C is wrong. Implies that it is necessary a necessary assumption. It doesn't state unless older people do not drink that it prevent the vitamin deficiency,which cause high blood pressure. If we saw may not that would fit sufficient condition, it would be a good contender.Also, we don't see that. However, we see the word "does not" in the answer choice implies a necessary condition. It means it definite and nothing else. We are looking for a condition that is a sufficient assumption. This answer choice doesn't fit what we are looking for.

    D is wrong and out of scope

    E is wrong much like D

    B is wrong because the quip talks about older people drinking milk to overcome the deficiency to prevent high pressure. B talks about milk containing no substance is the cause of the deficiency. No place is assumed in the paragraph that it does that.

    Now, let try try A. In the paragraph, there are two words you can zero on the word any and some. If you see the word some. This tells me that is that we looking an answer that implies that it is a sufficient assumption. It talks about in the second to last premise that one glass can easily make up any calcium deficiency. Any is just another way of saying if. Implies a sufficient condition. Also, have modifier "can",which is indicates a sufficient condition. A is correct.

Sign In or Register to comment.